1answer.
Ask question
Login Signup
Ask question
All categories
  • English
  • Mathematics
  • Social Studies
  • Business
  • History
  • Health
  • Geography
  • Biology
  • Physics
  • Chemistry
  • Computers and Technology
  • Arts
  • World Languages
  • Spanish
  • French
  • German
  • Advanced Placement (AP)
  • SAT
  • Medicine
  • Law
  • Engineering
sattari [20]
2 years ago
5

Hello, Brainly community!

Mathematics
1 answer:
ioda2 years ago
8 0

Answer:

(B)  \displaystyle \frac{W(3.1) - W(2.9)}{0.2}

General Formulas and Concepts:

<u>Calculus</u>

Limits

Derivatives

  • The definition of a derivative is the slope of the tangent line.

Derivative Notation

Instantaneous Rates

  • Tangent Line: \displaystyle f'(x) = \frac{f(b) - f(a)}{b - a}

Step-by-step explanation:

Since we are trying to find a <em>rate</em> at which W(t) changes, we must find the <em>derivative</em> at <em>t</em> = 3.

We are given 2 close answer choices that would have the same <em>numerical</em> answer but different <em>meanings</em>:

  1. (A)  \displaystyle  \lim_{t \to 3} W(t)
  2. (B)  \displaystyle \frac{W(3.1) - W(2.9)}{0.2}

If we look at answer choice (A), we see that our units would simply just be volume. It would not have the units of a rate of change. Yes, it may be the closest numerically correct answer, but it does not tell us the <em>rate</em> at which the volume would be changing and it is not a derivative.

If we look at answer choice (B), we see that our units would be cm³/s, and that is most certainly a rate of change. Answer choice (B) is also a <em>derivative</em> at <em>t</em> = 3, and a derivative tells us what <em>rate</em> something is changing.

∴ Answer choice (B) will give us the best estimate for the value of the instantaneous rate of change of W(t) when <em>t</em> = 3.

Topic: AP Calculus AB/BC (Calculus I/I + II)

Unit: Differentiation

Book: College Calculus 10e

You might be interested in
help plz plz plz plz plz plz plz plz plz plz plz plz plz plz plz plz plz plz plz plz plz plz plz plz plz plz plz plz plz plz plz
Rudiy27

Answer:

Q. 4.

-> $3,333

Q. 5

-> $40,000

3 0
2 years ago
Read 2 more answers
A carton of eggs costs $2.90 and a package of bacon costs $3.10. How much
Dominik [7]

Answer:

$8.90

Step-by-step explanation:

((2)(2.90))+(3.10)= $8.90

3 0
3 years ago
Read 2 more answers
Which of the following functions is not a sinusoid?
Nezavi [6.7K]
A sinusoid is cyclical and has a period or frequency.

y = sin x is a sinusoid, with a period of 2π.
y = √x is not a sinusoid.
y = cos x is a sinusoid with a period of 2π.

Answer: y = √x is not a sinusoid.
8 0
3 years ago
Read 2 more answers
Write the recurring decimal 0.1 as a fraction in its simplest form
IrinaVladis [17]
The answer is going to be 1/9
6 0
3 years ago
Read 2 more answers
ASAPPP HELP<br> PLEASE!!!!!!
Nostrana [21]

Answer:

-36/7

Step-by-step explanation:

-4/7 x 9

-4 x 9/ 7

That’s how I got -36/7

7 0
1 year ago
Other questions:
  • What is the factored form of 4x^2+23x-72
    10·1 answer
  • If Travis tries to drink 3 quarts of water each day how many cups of water does he drink each day
    9·1 answer
  • How do I find the value of an expression ​
    8·1 answer
  • I need an example of ‘what is a 3rd degree polynomial with two terms’ ?
    9·1 answer
  • PLEASE HELP!!!!!!<br><br> I really need help on this! It's due today and I'm kinda late on it!!
    7·1 answer
  • Answer ASAP I am being timed
    12·1 answer
  • Consider the expression below.
    15·2 answers
  • In a survey of a community, it was found that 85% of the people like winter season and 65% like summer season. If none of them d
    14·1 answer
  • Rewrite each equation below so that it is solved for y.
    5·1 answer
  • Triangle ABC was dilated by 50%. AC 6 what is AC?<br> A) 3<br> B) 6<br> c) 12<br> d) 15
    13·2 answers
Add answer
Login
Not registered? Fast signup
Signup
Login Signup
Ask question!